数学人眼中的湖北(五)

**数学人眼中的湖北(五)**

随着`武汉`解封,越来越多的朋友实现了团聚,那些奋战在一线夜以继日工作的医护工作者们也能松一口气,感谢他们的辛苦奉献,让我们安心在家隔离,放心学习和工作。今天继续给大家带来`李yt`博士对湖北高考题的精彩赏析,探究高考命题背景与规律,喜欢钻研并希望提高不等式与导数解题技巧,`在高考中取得高分`的朋友赶紧看过来。

对于某些不等式, 尤其是左边是数列前$n$项的和式(积式), 右边是某个常数,
我们应用归纳法, 一般来说是没有办法利用归纳假设做递推的,
这个时候, 需要待证不等式的右边进行改造和加强. 下面以2015年湖北理科数学压轴题为例.

(2015年湖北卷, 理科第22题)已知数列${a_n}$的各项均为正数, 且$displaystyle b_n=nleft(1+frac{1}{n} ight)^na_n(nin mathbb{N})$,其中$e$为自然对数的底数.

(1) 求函数$f(x)=1+x-e^x$的单调区间, 并比较$displaystyle left(1+frac{1}{n} ight)^n$与$e$的大小.

(2) 计算$displaystyle frac{b_1}{a_1},frac{b_1b_2}{a_1a_2},frac{b_1b_2b_3}{a_1a_2a_3}$, 由此推测计算
$displaystyle frac{b_1b_2cdots b_n}{a_1a_2cdots a_n}$的公式, 并给出证明.

(3) 令$displaystyle c_n=(a_1a_2cdots a_n)^{frac{1}{n}}$, 数列${a_n}$和${c_n}$的前$n$项和分别记为
$S_n,T_n$, 试证明: $ T_n<eS_n$.


我们下面仅仅说明第(3)小问, 即需要证明
egin{equation*} label{e137} sumlimits_{k=1}^nsqrt[k]{a_1a_2cdots a_k}
leqslant esumlimits_{k=1}^na_k,quad (1) end{equation*}
上式称为Carleman不等式, 进一步, 我们还可以证明, 右端的常数$e$是最佳的,
也就是说, 没有更小的常数使得上式成立.

**证明一.** 令$b_k=dfrac{(k+1)^k}{k^{k-1}}(k=1,2,ldots, n)$, 根据AM-GM不等式可得
egin{equation*}sumlimits_{k=1}^nsqrt[k]{a_1a_2cdots a_k}
=sumlimits_{k=1}^nfrac{sqrt[k]{(a_1b_1)(a_2b_2)cdots (a_kb_k)}}{k+1}
leqslant sumlimits_{k=1}^nfrac{1}{k(k+1)}left( sumlimits_{i=1}^na_ib_i ight) end{equation*}
egin{equation*}= sumlimits_{i=1}^n a_ib_isumlimits_{j=i}^nfrac{1}{j(j+1)}
= sumlimits_{i=1}^n a_ib_i left( frac{1}{i}-frac{1}{n+1} ight).end{equation*}
注意到
egin{equation*} b_i left( frac{1}{i}-frac{1}{n+1} ight)<frac{b_i}{i}=left( 1+frac{1}{i} ight)^i< e.end{equation*}
因此, 不等式(1)成立.

我们接下来说明$e$是最佳的, 令$a_k=1/k$, 于是
egin{equation*} limlimits_{n o infty}{sumlimits_{k=1}^nfrac{1}{k}}iggm/ {sumlimits_{k=1}^n
frac{1}{sqrt[n]{n!}}}=limlimits_{n o infty}frac{sqrt[n]{n!}}{n}=e.end{equation*}
所以, $e$是使得不等式(1)成立的最小常数.


**证明二.**
证明一构造的数列$b_k$不是唯一的, 注意到
egin{equation*}sqrt[k]{a_1a_2cdots a_k}=frac{1}{sqrt[k]{k!}}cdot sqrt[k]{(1cdot a_1)(2cdot a_2)cdots
(kcdot a_k)}leqslant frac{1}{sqrt[k]{k!}}cdot frac{a_1+2a_2+cdots +ka_k}{k}.end{equation*}
结合不等式$sqrt[n]{n!}>(n+1)/e$(见数学归纳法章节), 于是
egin{equation*} sumlimits_{k=1}^nsqrt[k]{a_1a_2cdots a_k}<ecdot sumlimits_{k=1}^n
frac{a_1+2a_2+cdots +ka_k}{k(k+1)}=ecdot sumlimits_{k=1}^n left(
sumlimits_{j=k}^nfrac{k}{j(j+1)} ight)a_k<esumlimits_{k=1}^na_k.quadsquare end{equation*}

**证明三.**
根据Hardy-Landau不等式(不懂就百度吧), 我们有
egin{equation*}sumlimits_{k=1}^nleft( frac{a_1^{1/p}+a_2^{1/p}+cdots +a_k^{1/p}}{k} ight)^p
leqslant left( frac{p}{p-1} ight)^psumlimits_{k=1}^na_k.end{equation*}
我们令$p o +infty$得
egin{equation*}limlimits_{p o +infty}left( frac{a_1^{1/p}+a_2^{1/p}+cdots +a_k^{1/p}}{k} ight)^p
=sqrt[k]{a_1a_2cdots a_k},end{equation*}
以及
egin{equation*} limlimits_{p o +infty}left( frac{p}{p-1} ight)^p=e. end{equation*}
所以, 不等式(1)成立.

**注.** 对于AM-GM不等式, 我们在利用它放缩时, 通常会对它加以调整, 通过引入待定的参数使得不等式更加精确.
比如, 当$lambda_k>0$时,
egin{equation*}sqrt[n]{a_1a_2cdots a_n}=frac{sqrt[n]{(lambda_1a_1)(lambda_2a_2)
cdots (lambda_na_n)}}{sqrt[n]{lambda_1lambda_2cdots lambda_n}}
leqslant frac{frac{1}{n}sumlimits_{k=1}^nlambda_ka_k}{sqrt[n]{
lambda_1lambda_2cdots lambda_n}}.end{equation*}
特别地, 取$lambda_k=k$时有
egin{equation*}sqrt[n]{a_1a_2cdots a_n}=frac{sqrt[n]{a_1(2a_2)cdots (na_n)}}{sqrt[n]{n!}}
leqslant frac{1}{sqrt[n]{n!}}cdot frac{1}{n}sumlimits_{k=1}^nka_k.end{equation*}

对于第(3)小问, 其难度是相当大的, 远远超出了中学生的能力范围, 属于竞赛类选手的难度.
直接证明不等式(1)比较不容易, 但是我们对其右端改造后, 就可以很轻松地利用数学归纳法,
对, 确实是很轻松.

**命题1.** 设$a_1,a_2,ldots ,a_n$为非负实数, $e$为自然对数的底数, 则
egin{equation*}label{e138} sumlimits_{k=1}^nsqrt[k]{a_1a_2cdots a_k}
leqslant esumlimits_{k=1}^na_k - nsqrt[n]{a_1a_2cdots a_n}.end{equation*}

**证明.** 利用归纳法就等价于证明
egin{equation*} ea_n+(n-1)sqrt[n-1]{a_1a_2cdots a_{n-1}}geqslant (n+1)sqrt[n]{a_1a_2cdots a_n}.end{equation*}
由AM-GM不等式可得
egin{equation*} ea_n+(n-1)sqrt[n-1]{a_1a_2cdots a_{n-1}}geqslant nsqrt[n]{ea_1a_2cdots a_n}.end{equation*}
再结合不等式$e>left( 1+frac{1}{n} ight)^n$即可.

当然, 这样的例子还有很多, 数学归纳法起着很巧妙绝伦的作用.

**命题2.** (羊明亮)设$x_1,x_2,ldots ,x_n$为任意实数, 证明:
egin{equation*}sumlimits_{k=1}^nleft( frac{1}{k}sumlimits_{j=1}^kx_j ight)^2leqslant
sumlimits_{k=1}^n(k+1)x_k^2.end{equation*}

**注:** 同样地, 利用归纳法可以证明如下不等式.
egin{equation*}sumlimits_{k=1}^nleft( frac{1}{k}sumlimits_{j=1}^kx_j ight)^2 leqslant sumlimits_{k=1}^n(k+1)x_k^2 - frac{1}{n}left(sumlimits_{k=1}^nx_k ight)^2.end{equation*}


**命题3.** (2005年国家队选拔赛试题)
设$a_1,a_2,ldots ,a_n$为正实数, 试证明:
egin{equation*}left( frac{sumlimits_{j=1}^nsqrt[j]{a_1a_2cdots a_j}}{sumlimits_{j=1}^na_j} ight)^{!!1/n}
+frac{sqrt[n]{a_1a_2cdots a_n}}{sumlimits_{j=1}^nsqrt[j]{a_1a_2cdots a_j}}leqslant
frac{n+1}{n}.end{equation*}

**注:** 该不等式是Carleman不等式的加强(为什么).


下面几例是类似的交换求和顺序的技巧.

**命题4.** 给出最佳常数$C$, 使得对任意正数$a_1,a_2,ldots,a_n$, 下述不等式成立.
egin{equation*} sumlimits_{k=1}^nfrac{k}{sumlimits_{j=1}^kfrac{1}{a_j}}leqslant Csumlimits_{k=1}^na_k.end{equation*}

**证明.** 根据Cauchy-Schwarz不等式可得
egin{equation*}left( sumlimits_{j=1}^kfrac{1}{a_j} ight)left( sumlimits_{j=1}^kj^2a_j ight)geqslant
left( sumlimits_{j=1}^kj ight)^2=left[frac{k(k+1)}{2} ight]^2.end{equation*}
整理便有
egin{equation*} frac{k}{sumlimits_{j=1}^kfrac{1}{a_j}}leqslant frac{4}{k(k+1)^2}left( sumlimits_{j=1}^kj^2a_j ight).end{equation*}
所以
egin{equation*}sumlimits_{k=1}^nfrac{k}{sumlimits_{j=1}^kfrac{1}{a_j}}
leqslant sumlimits_{k=1}^nfrac{4}{k(k+1)^2}left( sumlimits_{j=1}^kj^2a_j ight) end{equation*}
egin{equation*}=2sumlimits_{j=1}^nj^2a_jsumlimits_{k=j}^nfrac{2}{k(k+1)^2}end{equation*}
egin{equation*}<2sumlimits_{j=1}^nj^2a_jsumlimits_{k=j}^nleft( frac{1}{k^2}-frac{1}{(k+1)^2} ight) end{equation*}
egin{equation*}=2sumlimits_{j=1}^nj^2a_jleft[ frac{1}{j^2}-frac{1}{(n+1)^2} ight]<2sumlimits_{j=1}^na_j.end{equation*}
因此, 可取$C=2$, 下面说明$2$是最佳系数. 令$a_j=1/j\,(j=1,2,cdots,n)$,
egin{equation*} sumlimits_{k=1}^nfrac{k}{sumlimits_{j=1}^kfrac{1}{a_j}}=sumlimits_{k=1}^n
frac{k}{frac{1}{2}k(k+1)}=2sumlimits_{k=1}^nfrac{1}{k+1}leqslant Csumlimits_{k=1}^nfrac{1}{k}.end{equation*}
由极限理论可知, 常数$C$不小于$2$.

令$a_i=1/x_i$, 命题即为2005年美国数学月刊上第11145号征解问题:

**例.** (AMM, 11145)
设$x_1,x_2,ldots ,x_n$均为正数, 试证明:
egin{equation*} sumlimits_{k=1}^nfrac{k}{x_1+x_2+cdots +x_k}
leqslant 2sumlimits_{k=1}^nfrac{1}{x_k}.end{equation*}


**注.**
从该不等式可以看出

(1) 若$x_k>0$, 且$displaystyle sumlimits_{n=1}^{infty}frac{1}{x_n}$收敛,
则级数
$displaystyle sumlimits_{n=1}^{infty}frac{n}{x_1+x_2+cdots +x_n}$
也是收敛的.

(2)下面这个不等式留给读者.
egin{equation*} sumlimits_{k=1}^nfrac{2k+1}{x_1+x_2+cdots +x_k}
leqslant 4sumlimits_{k=1}^nfrac{1}{x_k}.end{equation*}

**命题5.**
设$a_kgeqslant 0\,(k=1,2,ldots ,n)$, 则对每个正整数$m$有
egin{equation*} sumlimits_{k=1}^nsqrt[k]{a_1a_2cdots a_k}leqslant frac{1}{m}sumlimits_{k=1}^n
a_kleft( frac{k+m}{k} ight)^k.end{equation*}

原文地址:https://www.cnblogs.com/Eufisky/p/12667423.html